Spotting and fixing errors: sentence structure, punctuation, and agreement - PSAT writing and language

PSAT/NMSQT Prep 2020 - Princeton Review 2020

Spotting and fixing errors: sentence structure, punctuation, and agreement
PSAT writing and language

Learning Objectives

After completing this chapter, you will be able to:

· Determine the correct punctuation and/or conjunctions to form a complete sentence

· Identify and correct inappropriate uses of semicolons

· Identify and correct inappropriate uses of commas, dashes, and colons

· Use punctuation to set off simple parenthetical elements.

· Identify and correct verb agreement issues

· Identify and correct pronoun agreement issues

· Identify and correct modifier agreement issues

· Identify and correct inappropriate uses of apostrophes

· Identify and correct expressions that deviate from idiomatic English

· Determine the appropriate word in frequently confused pairs

145/600 SmartPoints®

How Much Do You Know?

Directions

Try the questions that follow. The “Category” heading in the explanation for each question gives the title of the lesson that covers how to answer it. If you answered the question(s) for a given lesson correctly, you may be able to move quickly through that lesson. If you answered incorrectly, you may want to take your time on that lesson.

1. Questions 1—11 refer to the following passage.

Economic Regulation

First introduced by Senator John Sherman of Ohio, the U.S. Congress passed the Sherman Antitrust Act in 1890. The Act made illegal “every contract, combination in the form of trust or otherwise, or conspiracy in the restraint of trade.” However, many critics of the time charged that the decidedly vague wording introduced by the pro-business senators who rewrote the act before its final approval results in the emasculation of the law’s anti-monopoly intent. Nevertheless, the Act was the first law to fight, even symbolically, against economic monopolies in the “open” market economy of the United States.

From the nation’s beginning, many politicians and influential, business leaders had maintained that the ideal economy in a democracy was one in which the government played a very limited role in regulating commerce. They argued that, by permitting businesses to pursue their own interests, the government actually promoted the interests of the nation as a whole. A quote often attributed to Charles E. Wilson, a former chairperson of General Motors, accurately captured the prevailing attitude of big business. “What’s good for General Motors is good for the nation.” Many of the leaders of trusts and monopolies in the 1800s co-opted the then cutting-edge terminology of Charles Darwin’s theory of natural selection, they argued that in an unrestrained economy, power and wealth would naturally flow to the most capable according to the principles of “social Darwinism.” Their monopolies were thus natural and efficient outcomes of economic development.

Toward the close of the 1800s, however, an increasingly large and vocal number of lower- and middle-class dissenters felt that these hands-off economic policies of the federal government allowed monopolies like Standard Oil to manipulate consumers by fixing prices, exploiting workers by cutting wages, and threaten democracy by corrupting politicians. Most directly, the trusts and monopolies completely destroyed the opportunities for competitors in their industries to do business effectively. The concerns of these working-class dissenters thus created a groundswell of support for the Sherman Antitrust Act, and attempted to outlaw these monopolies and trusts. Even more important then the direct effects of the Act, however, was the shift toward a new era of reform against monopolistic economic corruption and the rise of deliberate economic regulation in America. The federal government realizing finally that they had to take a more active role in the economy in order to protect the interests and rights of consumers, workers, and small businesses while tempering the dominating power of big business.

2.

1. NO CHANGE

2. In 1890, first introduced by Senator John Sherman of Ohio, the U.S. Congress passed the Sherman Antitrust Act.

3. The U.S. Congress passed the Sherman Antitrust Act, first introduced by Senator John Sherman of Ohio, in 1890.

4. The U.S. Congress, first introduced by Senator John Sherman of Ohio, passed the Sherman Antitrust Act in 1890.

3.

1. NO CHANGE

2. will result

3. resulted

4. has resulted

4.

1. NO CHANGE

2. influentially business leaders

3. influential business leaders

4. influential business, leaders

5.

1. NO CHANGE

2. Charles E. Wilson a former chairperson of General Motors

3. Charles E. Wilson—a former chairperson of General Motors

4. Charles E. Wilson a former chairperson of General Motors,

6. Which choice most effectively combines the two sentences at the underlined portion?

1. business, and

2. business;

3. business

4. business:

7.

1. NO CHANGE

2. arguing

3. they argue

4. to argue

8.

1. NO CHANGE

2. exploit

3. exploits

4. exploited

9.

1. NO CHANGE

2. Act, which were attempting

3. Act, which attempted

4. Act, and attempting

10.

1. NO CHANGE

2. against

3. and

4. than

11.

1. NO CHANGE

2. finally realizing

3. finally will have realized

4. had finally realized

12.

1. NO CHANGE

2. we

3. it

4. one

13.

Check Your Work

1. C

Difficulty: Medium

Category: Agreement: Modifiers

Getting to the Answer: Modifying phrases must be placed as close as possible to what they modify. The intended meaning is not that the U.S. Congress was “first introduced by Senator John Sherman of Ohio”; rather, the phrase describes the Sherman Antitrust Act. Only (C) correctly places the modifying phrase adjacent to the Act, so (C) is correct.

2. C

Difficulty: Easy

Category: Agreement: Verbs

Getting to the Answer: Unless the context in the passage indicates that the time frame has changed, verb tenses should be consistent. The context makes clear that everything described in this sentence happened in the past. Choice (C), which is in the simple past tense like the other verbs in the sentence (“charged” and “rewrote”), corrects the unnecessary shift in verb tense. The other answer choices are incorrect because they are in other verb tenses: (A) is in present tense, (B) is in future tense, and (D) indicates an action that occurred in the past but has present consequences.

3. C

Difficulty: Medium

Category: Agreement: Modifiers

Getting to the Answer: This question tests how to use the words “influential” and “business” to modify “leaders” in a way that maintains the intended meaning. In this context, “business” defines the type of “leaders.” You can test this by reversing the order of the modifiers: “business influential leaders” does not make logical sense. “Influential” is thus modifying “business leaders,” not just “leaders,” so no comma is needed between the modifiers. (C) is correct. (B) is incorrect because the phrase “business leaders” functions as a noun, so you need an adjective, not an adverb, to modify it.

4. A

Difficulty: Hard

Category: Sentence Structure: Commas, Dashes, and Colons

Getting to the Answer: Parenthetical information in a sentence must be set off by punctuation, while essential information does not require punctuation. This question tests whether the descriptive phrase “a former chairman of General Motors” is essential in this sentence. Although the phrase adds relevant information, removing the phrase still results in a sentence that makes logical sense: the reader still knows precisely who (Charles E. Wilson) is associated with the quote. Since the phrase is parenthetical, it should be set off by a pair of commas, dashes, or parentheses—(A) is correct. The other choices are incorrect because they either omit the necessary punctuation entirely, as in (B), or use only one punctuation mark instead of a pair, as in (C) and (D).

5. D

Difficulty: Medium

Category: Sentence Structure: Commas, Dashes, and Colons

Getting to the Answer: You must determine the correct punctuation to use between these two independent clauses. Since several of the answer choices are technically correct, evaluate the content of the sentences to determine the relationship between them. The first sentence refers to a “quote” and the second sentence is the actual quote. Since the first sentence introduces the quote, a colon is appropriate, so (D) is correct. (A) and (B) are technically correct ways to join independent clauses, but they are incorrect in this context, in which the first clause introduces the second. (C) results in a run-on.

6. B

Difficulty: Medium

Category: Sentence Structure: The Basics

Getting to the Answer: The underlined portion links two independent clauses, each containing a subject and verb (“Many . . . co-opted” and “they . . . argued”) and expressing a complete thought. As written, a comma without a FANBOYS conjunction between two independent clauses creates a run-on sentence; eliminate (A). None of the other answer choices contains a FANBOYS conjunction, so look for another way to correct the run-on. Since the comma is not underlined, look for the choice that makes the second clause dependent—in this case, choice (B).

7. B

Difficulty: Hard

Category: Agreement: Verbs

Getting to the Answer: Whenever you see a series, make sure that all of the items are in parallel form. This is a series of three verb phrases that describe what “monopolies” were allowed to do. The entire series begins with “to,” and the non-underlined verb phrases begin with present tense verbs: “manipulate consumers” and “threaten democracy.” As written, the second item is “exploiting workers,” so it is not parallel with “manipulate” and “threaten.” Choice (B) follows the pattern of the other verb phrases and is correct.

8. C

Difficulty: Hard

Category: Sentence Structure: The Basics

Getting to the Answer: Check underlined verbs for their subjects. Make sure that the sentence makes logical sense. Often, the subject of a verb can be found much earlier in the sentence. As written, the underlined segment seems to have “concerns” as its subject, but it doesn’t make logical sense that the “concerns” “attempted” anything. Reading carefully, you will see that the phrase provides additional information about the Sherman Antitrust Act and should be subordinate to the main clause. Choice (C) fixes the issue and correctly uses the verb “attempted” to match the singular noun “Sherman Antitrust Act.”

9. D

Difficulty: Easy

Category: Agreement: Idioms

Getting to the Answer: Analyze the sentence so you can determine which word is correct in context. The underlined word helps set up a comparison between the Act’s “direct effects” and “a new era of reform.” The correct word to use in a comparison phrase such as “more than” is than, so (D) is correct. Choice (A), then, which is frequently confused with than, refers to sequence or to causation in “if/then” statements. Neither (B) nor (C) is idiomatically correct when used with the comparison “more.”

10.D

Difficulty: Easy

Category: Sentence Structure: The Basics

Getting to the Answer: Read long sentences carefully to make sure they contain a subject and a predicate verb and express a complete thought. As written, this sentence is a fragment because it lacks a predicate verb for the subject “the federal government.” Replacing the underlined segment with (D) adds a predicate verb and forms a complete sentence. (B) fails to fix the initial error. While (C) results in a complete sentence, it is incorrect because its verb tense does not match the context.

11.C

Difficulty: Medium

Category: Agreement: Pronouns

Getting to the Answer: When you see an underlined pronoun, read around the underlined segment to identify its antecedent so you can determine whether they are in agreement. Reading from the beginning of the sentence, you can see that the pronoun “they” stands in for “government.” Although the government consists of many people, the noun “government” is singular, so (C) is correct.

Sentence Structure: The Basics

Learning Objectives

After this lesson, you will be able to:

· Determine the correct punctuation and/or conjunctions to form a complete sentence

· Identify and correct inappropriate uses of semicolons

To answer a question like this:

1. San Francisco’s cable cars get their name from the long, heavy cable that runs beneath the streets along which the cars travel, this cable system resembles a giant laundry clothesline with a pulley at each end. Electricity turns the wheels of the pulleys, which in turn make the cable move.

2.

1. NO CHANGE

2. travel and this cable

3. travel this cable

4. travel; this cable

You need to know this:

Fragments and run-ons

A complete sentence must have both a subject and a verb and express a complete thought. If any one of these elements is missing, the sentence is a fragment. You can recognize a fragment because the sentence will not make sense as written. There are some examples in the table below.

Missing Element

Example

Corrected Sentence

Subject

Ran a marathon.



Lola ran a marathon.

Verb

Lola a marathon.

Complete thought

While Lola ran a marathon.

While Lola ran a marathon, her friends cheered for her.

The fragment “While Lola ran a marathon” is an example of a dependent clause: it has a subject (Lola) and a verb (ran), but it does not express a complete thought because it starts with a subordinating conjunction (while). Notice what the word “while” does to the meaning: While Lola ran a marathon, what happened? To fix this type of fragment, eliminate the subordinating conjunction or join the dependent clause to an independent clause using a comma. Subordinating conjunctions are words and phrases such as since, because, therefore, unless, although, and due to.

Unlike a dependent clause, an independent clause can stand on its own as a complete sentence. If a sentence has more than one independent clause, those clauses must be properly joined. If they are not, the sentence is a run-on: Lucas enjoys hiking, he climbs a new mountain every summer. There are several ways to correct a run-on as shown in the following table.

To Correct a Run-on

Example

Use a period

Lucas enjoys hiking. He climbs a new mountain every summer.

Use a semicolon

Lucas enjoys hiking; he climbs a new mountain every summer.

Use a colon

Lucas enjoys hiking: he climbs a new mountain every summer.

Make one clause dependent

Since Lucas enjoys hiking, he climbs a new mountain every summer.

Add a FANBOYS conjunction: For, And, Nor, But, Or, Yet, So

Lucas enjoys hiking, so he climbs a new mountain every summer.

Use a dash

Lucas enjoys hiking—he climbs a new mountain every summer.

Semicolons

Semicolons are used in two specific ways:

· A semicolon may join two independent clauses that are not connected by a FANBOYS conjunction (also called a coordinating conjunction), just as you would use a period.

· Semicolons may be used to separate items in a list if those items already include commas.

Use semicolons to . . .

Example

Join two independent clauses that are not connected by a comma and FANBOYS conjunction

Gaby knew that her term paper would take at least four hours to write; she got started in study hall and then finished it at home.

Separate sublists within a longer list when the sub-lists contain commas

The team needed to bring uniforms, helmets, and gloves; oranges, almonds, and water; and hockey sticks, pucks, and skates.

You need to do this:

To recognize and correct errors involving fragments, run-ons, and semicolons, familiarize yourself with the ways in which they are tested.

· Fragments

o If a sentence is missing a subject, a verb, or a complete thought, it is a fragment.

o Correct the fragment by adding the missing element.

· Run-ons

o If a sentence includes two independent clauses, they must be properly joined.

o Employ one of the following options to properly punctuate independent clauses:

§ Use a period

§ Insert a semicolon

§ Use a comma and a FANBOYS (for, and, nor, but, or, yet, so) conjunction

§ Use a dash

§ Make one clause dependent by using a subordinating conjunction (since, because, therefore, unless, although, due to, etc.)

· Semicolons

o A semicolon is used to join two independent clauses that are not connected by a comma and FANBOYS conjunction.

o Semicolons separate sublists within a longer list. (The items inside the sublists are separated by commas.)

Explanation:

If a clause could stand alone as a complete sentence, it is independent. As written, the sentence is a run-on because two independent clauses cannot be joined by only a comma, so (A) is incorrect. Only (D) corrects the run-on by joining the independent clauses with a semicolon. (B) is incorrect because the FANBOYS conjunction “and” must be preceded a comma to join independent clauses. (C) is incorrect because it eliminates all punctuation and is still a run-on.

If sentence formation or semicolons give you trouble, study the preceding information and try these Drill questions before completing the Try on Your Own questions that follow. Answers to the Drill can be found below.

Drill

1. Correct the fragment by adding a subject: Drove to the store to buy ice cream.

2. Correct the fragment by completing the thought: Despite arriving late to the movie.

3. Correct the run-on sentence with a punctuation mark: I hope that Zahra can attend the study session she has a gift for clearly explaining geometry questions.

4. Correct the run-on sentence with a conjunction: Visiting Washington, D.C., is a great experience because you can immerse yourself in the nation’s political history, another perk is the free admission at the Smithsonian museums.

5. Correct the run-on sentence by making one clause dependent: The early computer ENIAC could make only simple computations, it was still a landmark achievement.

Drill answers

Note: These are not the only ways to correct the sentences; your answers may differ.

1. Harold drove to the store to buy ice cream.

2. Despite arriving late to the movie, I still understood the plot.

3. I hope that Zahra can attend the study session; she has a gift for clearly explaining geometry questions.

4. Visiting Washington, D.C., is a great experience because you can immerse yourself in the nation’s political history, and another perk is the free admission at the Smithsonian museums.

5. Although the early computer ENIAC could make only simple computations, it was still a landmark achievement.

Try on Your Own

Directions

Take as much time as you need on these questions. Work carefully and methodically. There will be an opportunity for timed practice at the end of the chapter.

1. Questions 2—6 refer to the following passage.

Penicillin

In 1928, bacteriologist Dr. Alexander Fleming observed that a spot of mold had contaminated one of the glass plates on which he was growing a colony of bacteria. Since he did not discard the plate immediately. He noticed that bacteria were flourishing everywhere on the plate except in the mold’s vicinity. He decided to culture the mold; and found that a broth filtered from it inhibited the growth of several species of bacteria. Nine years later, a team of scientists led by Howard Florey and Ernst Chain isolated the active antibacterial agent in Fleming’s broth: penicillin. Florey and Chain went on to demonstrate that penicillin could cure bacterial infections in mice and in humans. Penicillin became a “miracle drug.”

Since these discoveries, medical specialists have prescribed penicillin to effectively combat bacterial infections, but problems concerning usage of this antibiotic have begun to emerge. Some people are allergic to penicillin. Though the number of those who are truly allergic is probably low. Side effects to the antibiotic are more frequent and include common reactions such as nausea; rash; and vomiting; less-common reactions such as fever; wheezing; and irregular breathing; and rare, life-threatening reactions such as anaphylaxis and seizures. However, the most significant problem with penicillin usage is the increasing prevalence of bacteria that are becoming penicillin-resistant, these bacteria cannot be effectively treated with current antibiotic strains. Despite these difficulties, careful penicillin administration will continue to save lives and reduce suffering from medical conditions.

2.

1. NO CHANGE

2. immediately and he

3. immediately, he

4. immediately; he

3.

1. NO CHANGE

2. mold and found

3. mold, and found

4. mold. And found

4.

1. NO CHANGE

2. penicillin,

3. penicillin, and though

4. penicillin, though

5.

1. NO CHANGE

2. nausea, rash, and vomiting, uncommon reactions such as fever, wheezing, and irregular breathing, and

3. nausea, rash, and vomiting, uncommon reactions such as fever, wheezing, and irregular breathing; and

4. nausea, rash, and vomiting; uncommon reactions such as fever, wheezing, and irregular breathing; and

6.

1. NO CHANGE

2. penicillin-resistant; these

3. penicillin-resistant these

4. penicillin-resistant; although these

7.

Sentence Structure: Commas, Dashes, and Colons

Learning Objectives

After this lesson, you will be able to:

· Identify and correct inappropriate uses of commas, dashes, and colons

· Use punctuation to set off simple parenthetical elements

To answer a question like this:

1. San Francisco’s famous cable cars are not powered and don’t generate any locomotion. Instead, each car has a powerful claw under its floor. The claw grips the cable when the car is ready to move, and releases the cable when the car needs to stop. The cars simply cling to the cable, which pulls them up and down San Francisco’s steep hills.

2.

1. NO CHANGE

2. move and releases

3. move; and releases

4. move—and releases

You need to know this:

Answer choices often move punctuation marks around, replace them with other punctuation marks, or remove them altogether. When underlined portions include commas, dashes, or colons, check to make sure the punctuation is used correctly in context.

Commas

There are two ways in which commas are not interchangeable with any other punctuation: a series of items and introductory words or phrases.

Use commas to...

Comma(s)

Set off three or more items in a series

Jeremiah packed a sleeping bag, a raincoat, and a lantern for his upcoming camping trip.

Separate an introductory word or phrase from the rest of the sentence

For example, carrots are an excellent source of several vitamins and minerals.

Commas and dashes

In many cases, either a comma or a dash may be used to punctuate a sentence.

Use commas or dashes to...

Comma(s)

Dash(es)

Separate independent clauses connected by a FANBOYS conjunction (For, And, Nor, But, Or, Yet, So)

Jess finished her homework earlier than expected, so she started an assignment that was due the following week.

Jess finished her homework earlier than expected—so she started an assignment that was due the following week.

Separate an independent and dependent clause

Tyson arrived at school a few minutes early, which gave him time to organize his locker before class.

Tyson arrived at school a few minutes early—which gave him time to organize his locker before class.

Separate parenthetical elements from the rest of the sentence (use either two commas or two dashes, not one of each)

Professor Mann, who is the head of the English department, is known for assigning extensive projects.

Professor Mann—who is the head of the English department—is known for assigning extensive projects.

Colons and dashes

Colons and dashes are used to include new ideas by introducing or explaining something, or by breaking the flow of the sentence. Note that the clause before the colon or dash must be able to stand on its own as a complete sentence.

Use colons and dashes to...

Colon

Dash

Introduce and/or emphasize a short phrase, quotation, explanation, example, or list

Sanjay had two important tasks to complete: a science experiment and an expository essay.

Sanjay had two important tasks to complete—a science experiment and an expository essay.

Separate two independent clauses when the second clause explains, illustrates, or expands on the first sentence

Highway 1 in Australia is one of the longest national highways in the world: it circles the entirety of the continent and connects every mainland state capital.

Highway 1 in Australia is one of the longest national highways in the world—it circles the entirety of the continent and connects every mainland state capital.

Unnecessary punctuation

Knowing when punctuation should not be used is equally important. If an underlined portion includes punctuation, take time to consider if it should be included at all.

Do NOT use punctuation to...

Incorrect

Correct

Separate a subject from its verb

The diligent student council, meets every week.

The diligent student council meets every week.

Separate a verb from its object or a preposition from its object

The diligent student council meets, every week.

The diligent student council meets every week.

Set off elements that are essential to a sentence’s meaning

The, diligent student, council meets every week.

The diligent student council meets every week.

Separate adjectives that work together to modify a noun

The diligent, student council meets every week.

The diligent student council meets every week.

Parenthetical elements

Parenthetical elements may appear at the beginning, in the middle, or at the end of a sentence. They must be properly punctuated with parentheses, commas, or dashes for the sentence to be grammatically correct. A phrase such as the capital of France is considered parenthetical if the rest of the sentence is grammatically correct when it is removed. Do not mix and match; a parenthetical element must begin and end with the same type of punctuation.

Parenthetical Element Placement

Parentheses

Comma(s)

Dash(es)

Beginning

N/A

The capital of France, Paris is a popular tourist destination.

N/A

Middle

Paris (the capital of France) is a popular tourist destination.

Paris, the capital of France, is a popular tourist destination.

Paris—the capital of France—is a popular tourist destination.

End

A popular tourist destination is Paris (the capital of France).

A popular tourist destination is Paris, the capital of France.

A popular tourist destination is Paris—the capital of France.

You need to do this:

If the underlined portion includes punctuation, ask yourself:

· Is the punctuation used correctly?


o The punctuation needs to be the correct type (comma, dash, or colon) and in the correct location.

· Is the punctuation necessary?

o If you cannot identify a reason why the punctuation is included, the punctuation should be removed.

Explanation:

Make sure every underlined punctuation mark is serving a function. Commas serve a variety of purposes, but in this sentence as written, the underlined comma separates the subject (“claw”) from part of its compound verb (“grips . . . and releases”). A comma should never separate a subject and verb, so eliminate (A). Only (B) correctly punctuates the sentence by removing the punctuation altogether. The other choices also separate the subject and verb, punctuating the sentence with a semicolon or dash as though joining two independent clauses.

If commas, dashes, and colons give you trouble, study the information above and try these Drill questions before completing the Try on Your Own questions that follow. Edit each sentence to correct the punctuation issue. Answers to the Drill can be found below.

Drill

1. Jamal doesn’t plan to carve a jack-’o-lantern but he still had fun picking a pumpkin at the pumpkin patch.

2. Eleanor Roosevelt the longest serving First Lady of the United States considered her work on the United Nations’ Declaration of Human Rights one of her greatest accomplishments.

3. I have three final exams this week Statistics, Biology, and World Literature.

4. The legendary entertainer, Johnny Carson, hosted his late-night talk show for thirty years.

5. Enabling agriculture due to its annual flooding the Nile River was truly the source of life in ancient Egypt.

Drill answers

Note: These are not the only ways to correct the sentences; your answers may differ.

1. Jamal doesn’t plan to carve a jack-’o-lantern, but he still had fun picking a pumpkin at the pumpkin patch.

2. Eleanor Roosevelt, the longest serving First Lady of the United States, considered her work on the United Nations’ Declaration of Human Rights one of her greatest accomplishments. OR Eleanor Rooseveltthe longest serving First Lady of the United Statesconsidered her work on the United Nations’ Declaration of Human Rights one of her greatest accomplishments.

3. I have three final exams this week: Statistics, Biology, and World Literature.

4. The legendary entertainer Johnny Carson hosted his late-night talk show for thirty years. (Commas deleted.)

5. Enabling agriculture due to its annual flooding, the Nile River was truly the source of life in ancient Egypt.

Try on Your Own

Directions

Take as much time as you need on these questions. Work carefully and methodically. There will be an opportunity for timed practice at the end of the chapter.

1. Questions 8—15 refer to the following passage.

The Sistine Chapel

One shudders to contemplate Michelangelo’s reaction if he were to gaze up today at the famous frescoes* he painted on the ceiling of the Sistine Chapel over four centuries ago. A practical man: he would no doubt be unsurprised by the effects of time and environment on his masterpiece. He would be philosophical about the damage wrought by mineral salts left behind when rainwater leaked through the roof. He would probably also accept the layers of dirt and soot from coal braziers that heated the chapel and from candles, and incense burned during religious functions. However, he would be appalled by the ravages recently inflicted on his work by restorers.

The Vatican restoration team reveled in inducing a jarringly colorful transformation of the frescoes with special, cleaning solvents and computerized analysis equipment. However, the restorers did not achieve this effect as they claimed merely by removing the dirt and animal glue (employed by earlier restorers to revive muted colors) from the frescoes: the team removed Michelangelo’s final touches as well. Gone from the ceiling is the quality of suppressed anger and thunderous pessimism so often commented upon by admiring scholars. That quality was not an artifact of grime, not a misleading monochrome imposed on the ceiling by time, for Michelangelo himself applied a veil of glaze to the frescoes to darken them after he had deemed his work too bright. The master would have felt compelled to add a few more layers of glaze had the ceiling radiated forth as it does now. The solvents of the restorers stripped away the shadows of the frescoes, reacted chemically with Michelangelo’s pigments, and ultimately produced hues the painter never intended for his art.

Of course, the restorers left open an avenue for the reversal of their progress toward color and brightness. Since the layers of animal glue were no longer there to serve as protection the atmospheric pollutants from the city of Rome gained direct access to the frescoes. Observers already noticed significant darkening in some of the restored work a mere four years after its completion. It remains to be seen whether the measure introduced to arrest this process: an extensive climate-control system—will itself have any long-term effect on the chapel’s ceiling.

* fresco: a style of painting on plaster using water-based pigments

2.

1. NO CHANGE

2. man he

3. man, he

4. man—he

3.

1. NO CHANGE

2. candles—and

3. candles. And

4. candles and

4.

1. NO CHANGE

2. special cleaning solvents and computerized, analysis equipment.

3. special cleaning solvents, and computerized analysis equipment.

4. special cleaning solvents and computerized analysis equipment.

5.

1. NO CHANGE

2. effect, as they claimed, merely

3. effect as they claimed, merely

4. effect, as they claimed merely

6.

1. NO CHANGE

2. frescoes, the team

3. frescoes and the team

4. frescoes the team

7.

1. NO CHANGE

2. stripped away the shadows of the frescoes reacted chemically with Michelangelo’s pigments, and ultimately produced hues the painter never intended for his art.

3. stripped away the shadows of the frescoes; reacted chemically with Michelangelo’s pigments; and ultimately produced hues the painter never intended for his art.

4. stripped away the shadows of the frescoes, reacted chemically with Michelangelo’s pigments; and ultimately produced hues the painter never intended for his art.

8.

1. NO CHANGE

2. protection, the

3. protection—the

4. protection; the

9.

1. NO CHANGE

2. process—an extensive climate-control system—will

3. process, an extensive climate-control system—will

4. process; an extensive climate-control system; will

10.

Agreement: Verbs

Learning Objectives

After this lesson, you will be able to:

· Identify and correct verb agreement issues

To answer a question like this:

1. The astronauts of Apollo 13 have performed a routine maintenance check on the ship’s equipment immediately before an explosion occurred that forced them to cancel a moon landing and greatly endangered the lives of the crew.

2.

1. NO CHANGE

2. will have performed

3. had performed

4. was performing

You need to know this:

1. Verb tense

Verb tense indicates when an action or state of being took place: past, present, or future. The tense of the verb must fit the context of the passage. Each tense can express three different types of action.

2.

Type of Action

Past

Present

Future

Single action occurring only once

Connor planted vegetables in the community garden.

Connor plants vegetables in the community garden.

Connor will plant vegetables in the community garden.

Action that is ongoing at some point in time

Connor was planting vegetables in the community garden this morning before noon.

Connor is planting vegetables in the community garden this morning before noon.

Connor will be planting vegetables in the community garden this morning before noon.

Action that is completed before some other action

Connor had planted vegetables in the community garden every year until he gave his job to Jasmine.

Connor has planted vegetables in the community garden since it started five years ago.

Connor will have planted vegetables in the community garden by the time the growing season starts.

Subject-verb agreement

A verb must agree with its subject in person and number:

· Person (first, second, or third)

o First: I ask a question.

o Second: You ask a question.

o Third: She asks a question.

· Number (singular or plural)

o Singular: The apple tastes delicious.

o Plural: Apples taste delicious.

The noun closest to the verb is not always the subject: The chair with the lion feet is an antique. The singular verb in this sentence, is, is closest to the plural noun feet. However, the verb’s actual subject is the singular noun chair, so the sentence is correct as written.

When a sentence includes two nouns, only the conjunction and forms a compound subject requiring a plural verb form:

· Plural: Saliyah and Taylor are in the running club.

· Singular: Either Saliyah or Taylor is in the running club.

· Singular: Neither Saliyah nor Taylor is in the running club.

Collective nouns are nouns that name entities with more than one member, such as group, team, and family. Even though these nouns represent more than one person, they are grammatically singular and require singular verb forms:

· The collection of paintings is one of the most popular art exhibits in recent years.

· The team looks promising this year.

Parallelism

Verbs in a list, a compound, or a comparison must be parallel in form.

Feature

Example

Parallel Form

A list

Chloe formulated a question, conducted background research, and constructed a hypothesis before starting the experiment.

3 simple past verb phrases

A compound

Hunting and fishing were essential to the survival of Midwestern Native American tribes such as the Omaha.

2 -ing verb forms

A comparison

Garrett enjoys sculpting as much as painting.

2 -ing verb forms

Note that parallelism may be tested using other parts of speech besides verbs. In general, any items in a list, compound, or comparison must be in parallel form. For example, if a list starts with a noun, the other items in the list must also be nouns; if it starts with an adjective, the other items must be adjectives, etc.

Incorrect

Correct

Naomi likes pumpkin pie and to drink coffee on chilly weekend afternoons.

Naomi likes pumpkin pie and coffee on chilly weekend afternoons.

or

Naomi likes to eat pumpkin pie and drink coffee on chilly weekend afternoons.

Which of the dogs is the most docile and better behaved?

Which of the dogs is the most docile and best behaved?

or

Which of the dogs is the more docile and better behaved?

1. You need to do this:

If the underlined portion includes a verb, check that the verb:

· Reflects the correct tense: does it fit the context?

· Agrees with the subject in person and number

· Is parallel in form with other verbs in a series, list, or compound if there is one in the sentence

Explanation:

Check underlined verbs to make sure they agree with their subjects and match the tense of the passage. The subject of the underlined verb is “astronauts,” so a plural verb is required; eliminate (D). Check the surrounding context to determine the correct tense. The actions of the sentence happened in the past (“occurred,” “endangered”), but this performance of a maintenance check happened before another past action in the sentence: the explosion. “Had performed,” is the appropriate way to indicate the sequence of these past actions, so (C) is correct. The other choices are not appropriate ways to express a single past action that happened before another past action.

If verbs give you trouble, study the information above and try the following Drill questions before completing the Try on Your Own questions that follow. Edit each sentence to correct the verb issue. Answers to the Drill can be found below.

Drill

1. The delicious flavors offered by the new ice cream shop (ensure/ensures) that many customers are typically lined up waiting to buy a scoop.

2. The manga club and the quiz team (meet/meets) in the student union on alternating Tuesdays.

3. Neither the teacher nor the student (was/were) adequately prepared for class.

4. By the time the toddler finally finished his dinner, everyone else (finished/had finished) eating dessert.

5. Katrina’s favorite activities at the amusement park include riding the wooden roller coasters, driving the bumper cars, and (eating/to eat) the caramel candy apples.

Drill answers

1. The delicious flavors offered by the new ice cream shop ensure that many customers are typically lined up waiting to buy a scoop.

2. The manga club and the quiz team meet in the student union on alternating Tuesdays.

3. Neither the teacher nor the student was adequately prepared for class.

4. By the time the toddler finally finished his dinner, everyone else had finished eating dessert.

5. Katrina’s favorite activities at the amusement park include riding the wooden roller coasters, driving the bumper cars, and eating the caramel candy apples.

Try on Your Own

Directions

Take as much time as you need on these questions. Work carefully and methodically. There will be an opportunity for timed practice at the end of the chapter.

1. Questions 17—22 refer to the following passage.

Woolly Mammoth

The woolly mammoth and the mastodon probably best capture the public’s current image of prehistoric Ice Age animals. Typically, these now-extinct, herbivorous precursors to the modern-day elephant was about 10 feet tall at the shoulders and weighed nearly 6,000 pounds.

Although paleontologists have discovered remains of many woolly mammoths over the years, none has been found better preserved than the “Jarkov Mammoth” found on Siberia’s Taimyr Peninsula in 1997. Soon after a 9-year-old boy out playing in the snowy hills first spotted the remains, scientists descended on the site. Then, after battling weeks of frigid weather and approximately 20,000 years’ worth of dense frost coating the entire body of the mammoth, the assembled team finally had completed a successful excavation. Important for numerous scientific reasons, the Jarkov Mammoth, in particular, has helped scientists settle a debate that has been raging for many years concerning the possible reasons behind the sudden extinction of these ancient giants.

Woolly mammoths roamed the cold northern plains of the globe for much of the last 2 million years, including most of the Ice Age that began roughly 70,000 years ago. Then, quite suddenly, 10,000 years ago, a time that corresponds with the end of the Ice Age, the mammoths disappeared. Scientific theories explaining this rapid extinction ranged from meteor showers pelting Earth to suggestions of massive volcanic eruptions. Today, however, partially due to evidence taken from the Jarkov Mammoth, the scientific community generally agree that these creatures died out from a combination of changing climate, hunting pressures from humans, and probably even disease. In fact, scientists consider it likely that the rising temperatures accompanying the end of the Ice Age will work against the evolutionary adaptations of the mammoths, including their signature woolly coats of dense fur. Indeed, the demise of the Jarkov Mammoth seems to have involved a deep patch of mud, perhaps a sign that these behemoths were unaccustomed to treading on increasingly softer ground.

2.

1. NO CHANGE

2. captured

3. will have captured

4. captures

3.

1. NO CHANGE

2. are

3. were

4. is

4.

1. NO CHANGE

2. completed

3. is completing

4. have completed

5.

1. NO CHANGE

2. suggesting massive volcanic eruptions.

3. theories of massive volcanic eruptions.

4. massive volcanic eruptions.

6.

1. NO CHANGE

2. agrees

3. agreeing

4. have agreed

7.

1. NO CHANGE

2. work

3. working

4. worked

8.

Agreement: Pronouns

Learning Objectives

After this lesson, you will be able to:

· Identify and correct pronoun agreement issues

To answer a question like this:

1. The public library is an invaluable treasure trove of the wisdom, research, drama, and wit of the ages, all available for easy access to eager patrons. Indeed, anyone with a card can borrow them free of charge.

2.

1. NO CHANGE

2. it

3. those

4. the library’s resources

You need to know this:

Pronoun forms

A pronoun is a word that takes the place of a noun. Pronouns can take three different forms, each of which is used based on the grammatical role it plays in the sentence.

Form

Pronouns

Example

Subjective: The pronoun is used as the subject.

I, you, she, he, it, we, they, who

Rivka is the student who will lead the presentation.

Objective: The pronoun is used as the object of a verb or a preposition.

me, you, her, him, it, us, them, whom

With whom will Rivka present the scientific findings?

Possessive: The pronoun expresses ownership.

my, mine, your, yours, his, her, hers, its, our, ours, their, theirs, whose

Rivka will likely choose a partner whose work is excellent.

Note that a pronoun in subjective form can, logically, be the subject in a complete sentence. Pronouns that are in objective form cannot.

When there are two pronouns or a noun and a pronoun in a compound structure, drop the other noun or pronoun to tell which form to use—for example: Leo and me walked into town. If you were talking about yourself only, you would say, “I walked into town,” not “Me walked into town.” Therefore, the correct form is subjective, and the original sentence should read: Leo and I walked into town.

Pronoun-antecedent agreement

A pronoun’s antecedent is the noun it logically represents in a sentence. If the noun is singular, the pronoun must be singular; if the noun is plural, the pronoun must be plural.

Antecedent

Incorrect

Correct

selection

The selection of books was placed in their designated location.

The selection of books was placed in its designated location.

Addison

Addison fed the giraffes all of the lettuce they had purchased.

Addison fed the giraffes all of the lettuce she had purchased.

sapling

The sapling, along with dozens of flowers, was relocated to where they would thrive.

The sapling, along with dozens of flowers, was relocated to where it would thrive.

student

If a student is confused, they should ask for clarification.

If a student is confused, he or she should ask for clarification.

Ambiguous pronouns

A pronoun is ambiguous if its antecedent is either missing or unclear. When you see an underlined pronoun, make sure you can identify the noun to which it refers.

Ambiguous Pronoun Use

Corrected Sentence

Anthony walked with Cody to the ice cream shop, and he bought a banana split.

Anthony walked with Cody to the ice cream shop, and Cody bought a banana split.

You need to do this:

If the underlined portion includes a pronoun, find the logical antecedent. If there is no clear antecedent, the pronoun is ambiguous and this error must be corrected. Then check that the pronoun:

· Uses the correct form

o If the pronoun is the subject of the sentence, use a subjective pronoun such as I, you, she, he, it, we, they, or who.

o If the pronoun is an object within the sentence, use an objective pronoun such as me, you, her, him, it, us, they, or whom.

o If the pronoun indicates possession, use a possessive pronoun such as my, mine, your, yours, his, her, hers, its, our, ours, their, theirs, or whose.

· Agrees with its antecedent

o A singular antecedent requires a singular pronoun; a plural antecedent requires a plural pronoun.

Explanation:

Every underlined pronoun on the PSAT must have a crystal-clear antecedent. In this case, although it is understood from the context that the writer means that people can borrow library materials, the pronoun “them” is ambiguous—indeed, its nearest antecedent is “eager patrons,” which is not what the writer intended to mean can be borrowed! Eliminate (A), as well as the other pronoun answer choices, since they all result in ambiguity. (D) is correct because it identifies precisely what can be borrowed.

If pronouns give you trouble, study the information above and try these Drill questions before completing the Try on Your Own questions that follow. Edit each sentence to correct the pronoun issue. Answers to the Drill can be found on the next page.

Drill

1. The manager let the employee go home an hour early because she was in a good mood.

2. My parents had a great surprise for my sister and (I/me): a visit to the beach.

3. The manager moved the display of vintage comic books from (its/their) location in the back of the store to the front.

4. Fai was able to convince (him/his) teacher to give the class no homework over the long weekend.

5. After purchasing (her/their) tickets, Jen watched the singers perform in the concert.

Drill answers

1. The manager let the employee go home an hour early because the manager was in a good mood.

2. My parents had a great surprise for my sister and me: a visit to the beach.

3. The manager moved the display of vintage comic books from its location in the back of the store to the front.

4. Fai was able to convince his teacher to give the class no homework over the long weekend.

5. After purchasing her tickets, Jen watched the singers perform in the concert.

Try on Your Own

Directions

Take as much time as you need on these questions. Work carefully and methodically. There will be an opportunity for timed practice at the end of the chapter.

1. Questions 24—29 refer to the following passage.

James Joyce

As we contemplate the state of literature in our modern era, it is difficult to resist a longing for the epic writers of eras gone by. At times, we take great pains to merely remember that there were once authors such as Homer, Dante, and Melville: authors whom were able to relate stories of heroic travels and struggles. They did not waste their time or ours with trivial affairs; they compelled you to mull over the great philosophical questions of all time in stories that stand up to readers’ repeated perusal from generation to generation. They took extensive care to depict accurately the best and worst aspects of human nature; strove to ensure that they would be enhancing, rather than degrading, the public’s intellect; and did not resort to tricks or devices in order to garner readership for their writings. In all these regards, past writers are firmly distinguished from many modern writers: the most notorious of these modern writers is the Irish novelist James Joyce.

While literary critics typically offer vastly different assessments of its quality after our readings of Ulysses, Joyce’s novel that retells Homer’s The Odyssey, my contention is that Joyce sullied the very form of the epic genre. Whereas The Odyssey was a great tale of a noble hero’s struggle against a seemingly insurmountable series of trials in order to restore order and honor to his household, his book is nearly the direct opposite. The protagonist is no hero: his actions are listless and forgettable, and his obsession with obscene and undignified behavior is virtually nauseating. And even more shamefully, Joyce wasted his talent; subject matter aside, the art of masterfully crafting words came naturally to him Sadly, it is the literary world’s loss that he was not born a few centuries earlier, when his talents could have been utilized in a more dignified manner.

2.

1. NO CHANGE

2. which

3. who

4. who’re

3.

1. NO CHANGE

2. me

3. one

4. us

4.

1. NO CHANGE

2. Readers

3. Such authors

4. He

5.

1. NO CHANGE

2. their

3. his or her

4. my

6.

1. NO CHANGE

2. their

3. Joyce’s

4. Homer’s

7.

1. NO CHANGE

2. himself

3. his own self

4. he

8.

Agreement: Modifiers

Learning Objectives

After this lesson, you will be able to:

· Identify and correct modifier agreement issues

· Identify and correct inappropriate uses of apostrophes

To answer a question like this:

1. Wind power development could potentially impact populations of several nocturnally migrating bird and bat species. During their seasonal migrations, large numbers fly through the mountainous landforms used for wind turbine locations.

2.

1. NO CHANGE

2. large amounts

3. many birds and bats

4. birds and bats

You need to know this:

A modifier is a word or phrase that describes, clarifies, or provides additional information about another part of the sentence. Modifier questions require you to identify the part of a sentence being modified and use the appropriate modifier in the proper place.

In order to be grammatically correct, the modifier must be placed as close to the word it describes as possible. Use context clues in the passage to identify the correct placement of a modifier; a misplaced modifier can cause confusion and is always incorrect on test day.

Note that a common way the PSAT tests modifiers is with modifying phrases at the beginning of a sentence. Just like any other modifier, the modifying phrase grammatically modifies whatever is right next to it in the sentence. For example, consider the sentence, “While walking to the bus stop, the rain drenched Bob.” The initial phrase, “While walking to the bus stop,” grammatically modifies “the rain,” creating a nonsense sentence; the rain can’t walk to the bus stop. The writer meant that Bob was walking to the bus stop, so the sentence should read, “While walking to the bus stop, Bob was drenched by the rain.”

Modifier/Modifying Phrase

Incorrect

Correct

nearly

Andre nearly watched the play for four hours.

Andre watched the play for nearly four hours.

in individual containers

The art teacher handed out paints to students in individual containers.

The art teacher handed out paints in individual containers to students.

A scholar athlete

A scholar athlete, maintaining high grades in addition to playing soccer were expected of Maya.

A scholar athlete, Maya was expected to maintain high grades in addition to playing soccer.

Adjectives and adverbs

Use adjectives only to modify nouns and pronouns. Use adverbs to modify everything else.

· Adjectives are single-word modifiers that describe nouns and pronouns: Ian conducted an efficient lab experiment.

· Adverbs are single-word modifiers that describe verbs, adjectives, or other adverbs: Ian efficiently conducted a lab experiment.

Note that nouns can sometimes be used as adjectives. For example, in the phrase “the fashion company’s autumn line,” the word “fashion” functions as an adjective modifying “company” and the word “autumn” functions as an adjective modifying “line.”

Comparative/superlative

When comparing similar things, use adjectives that match the number of items being compared. When comparing two items or people, use the comparative form of the adjective. When comparing three or more items or people, use the superlative form.

Comparative (two items)

Superlative (three or more items)

better, more, newer, older, shorter, taller, worse, younger

best, most, newest, oldest, shortest, tallest, worst, youngest

Possessive nouns and pronouns

Possessive nouns and pronouns indicate that something belongs to someone or something. In general, possessive nouns are written with an apostrophe, while possessive pronouns are not.

To spot errors in possessive noun or pronoun construction, look for . . .

Incorrect

Correct

Two nouns in a row

The professors lectures were both informative and entertaining.

The professor’s lectures were both informative and entertaining.

Pronouns with apostrophes

The book is her’s.

The book is hers.

Words that sound alike

The three friends decided to ride there bicycles to the park over they’re where their going to enjoy a picnic lunch.

The three friends decided to ride their bicycles to the park over there where they’re going to enjoy a picnic lunch.

Apostrophes

Use an apostrophe to . . .

Example

Indicate the possessive form of a single noun

My oldest sister’s soccer game is on Saturday.

Indicate the possessive form of a plural noun

My two older sisters’ soccer games are on Saturday.

Indicate a contraction (e.g., don’t, can’t)

They’ve won every soccer match this season.

Note that plural nouns are formed without an apostrophe.

Incorrect

Correct

Sting ray’s are cartilaginous fish related to shark’s.

Sting rays are cartilaginous fish related to sharks.

There are many carnival’s in this area every summer.

There are many carnivals in this area every summer.

To check whether it’s is appropriate, replace it in the sentence with it is or it has. If the sentence no longer makes sense, it’s is incorrect. The following sentence is correct:

The tree frog blends perfectly into its surroundings; when it holds still, it’s nearly invisible.

Note that its’ and its’s are never correct.

You need to do this:

If the underlined portion includes a modifier, determine whether the modifier:

· Is placed correctly

o Is it as near as possible to the word it logically modifies?

o If it is not in the correct place, where should it be moved?

· Agrees with the word or words it is describing

o Does the sentence require an adjective or an adverb?

o Does the noun or pronoun show proper possession?

If the underlined portion includes an apostrophe, make sure it correctly indicates either possession or a contraction. If an apostrophe is missing, select the answer choice that places it in the correct location.

Explanation:

Make sure introductory modifiers are modifying the correct items. The introductory modifying phrase in this sentence is “During their seasonal migrations,” which must modify what directly follows it; as written, “large numbers.” It does not make sense that “numbers” migrate (as though you can see the number 17 flying across the sky). Eliminate (A) and (B), which make the same error. Even though both (C) and (D) correct the error, only choice (C) retains the original intended meaning: that numerous birds and bats migrate.

If modifiers give you trouble, study the information in this section and try these Drill questions before completing the Try on Your Own questions that follow. Edit each sentence to correct the modifier or apostrophe issue. Answers to the Drill can be found on the page after the Drill.

Drill

1. The colorfully impressive plumage of the tropical birds helped make the aviary the most popular destination at the zoo.

2. Since the gym is remodeling the womens locker room, I had to change into my workout clothes at home.

3. The players on the baseball team all felt an immense sense of relief after a hard-fought victory over there long-time rivals.

4. When asked if they preferred reality programs or news documentaries, viewers reported that reality programs were most entertaining.

5. Although normally considered a children’s toy, yo-yo performers are highly skilled professionals who can flawlessly execute impressive tricks.

Drill answers

Note: These are not the only ways to correct the sentences; your answers may differ.

1. The colorful, impressive plumage of the tropical birds helped make the aviary the most popular destination at the zoo.

2. Since the gym is remodeling the women’s locker room, I had to change into my workout clothes at home.

3. The players on the baseball team all felt an immense sense of relief after a hard-fought victory over their long-time rivals.

4. When asked if they preferred reality programs or news documentaries, viewers reported that reality programs were more entertaining.

5. Although the yo-yo is normally considered a children’s toy, yo-yo performers are highly skilled professionals who can flawlessly execute impressive tricks.

Try on Your Own

Directions

Take as much time as you need on these questions. Work carefully and methodically. There will be an opportunity for timed practice at the end of the chapter.

1. Questions 31—36 refer to the following passage.

Madame Bovary

Some critics believe that Madame Bovary, the most famous novel by French author Gustave Flaubert, has a strange and subversive theme that undermines its own medium: in short, they say that Flaubert’s masterpiece of fiction is a cautionary tale about the dangers of reading novels. As evidence, they point to its unsympathetic protagonist, Emma Bovary, who lives in her books and romanticizes the simplest aspects of daily life—for example, eating rich food and buying expensive clothing—as well as her relationships. Emma cares only about her immediate physical gratification and material possessions rather than the well-being of her friends and family, becoming cruel, shortsighted, and constantly dissatisfied with real life. Her fantasies lead to her downfall: her relationship with her well-meaning but naive husband gradually disintegrates, her two adulterous affairs end in disaster, her constant borrowing leads her family to financial ruin, and her desire to die in a glorious dramatic fashion leads instead to an unexpectedly agonizing three days of death throes. She expects too much from life and is punished horribly for it.

Certainly Emma’s flawed personality, as well as her literary obsession, contributes to her downfall, but it is interesting to note that only Emma is the only character in the novel who habitually reads for pleasure. In fact, her husband spends the bulk of the story engaged in the mundane activities of daily life: running a business, tending to family members, maintaining the household. He is naive, admittedly, but happy—at least until Emma’s penchant for romance begins to interfere with his responsibilities. Therefore, there really are no other appropriate characters with whom to compare her, although we can point out that the characters making up the novel’s non-reading population tend to be more socially responsible than Emma. Perhaps Madame Bovary, then, was not meant to be a criticism of fiction itself; rather, Flaubert intended his novel to be a caution against allowing suggestible characters like Emma to have access to novels. The permissive environment that the Bovarys permit in their household contributes to their downfall and social ruin; the characters’ unwillingness to check Emma’s passions (and even their ignorance of the existence of such a problem) leads to the disintegration of their family.

2.

1. NO CHANGE

2. it’s

3. they’re

4. their

3.

1. NO CHANGE

2. Becoming cruel, shortsighted, and constantly dissatisfied with real life, friends and family do not affect Emma’s well-being as she cares only about her immediate physical gratification and material possessions.

3. Becoming cruel, shortsighted, and constantly dissatisfied with real life, Emma cares only about her immediate physical gratification and material possessions rather than the well-being of her friends and family.

4. Caring only about her immediate physical gratification and material possessions, Emma’s friends and family become cruel, shortsighted, and constantly dissatisfied with real life.

4.

1. NO CHANGE

2. gloriously dramatic fashion

3. dramatic glorious fashion

4. glorious dramatically fashion

5.

1. NO CHANGE

2. that Emma is the only character in the novel who habitually reads for pleasure

3. that Emma is the character in the only novel who habitually reads for pleasure

4. that Emma is the character in the novel who only habitually reads for pleasure

6.

1. NO CHANGE

2. comparatively

3. greater

4. most

7.

1. NO CHANGE

2. character’s

3. characters

4. characters’s

8.

Agreement: Idioms

Learning Objectives

After this lesson, you will be able to:

· Identify and correct expressions that deviate from idiomatic English

· Determine the appropriate word in frequently confused pairs

To answer a question like this:

1. The United States Geological Survey (USGS) is studying the distribution and flight patterns of birds and bats that migrate at night. Researchers analyze weather surveillance radar data not only to assess the responses of migrant birds to prominent landforms but also to generate a broad overview of spring and fall migration through the Appalachians.

2.

1. NO CHANGE

2. not to assess the responses of migrant birds to prominent landforms but also to generate

3. not only to assess the responses of migrant birds to prominent landforms but to generate

4. to not only assess the responses of migrant birds to prominent landforms but also to generate

You need to know this:

An idiom is a combination of words that must be used together to convey either a figurative or literal meaning. Idioms are tested in three ways:

1. Proper preposition use in context: The preposition must reflect the writer’s intended meaning.

1. She waits on customers.

2. She waits for the bus.

3. She waits with her friends.

2. Idiomatic expressions: Some words or phrases must be used together to be correct.

1. Simone will either bike or run to the park.

2. Neither the principal nor the teachers will tolerate tardiness.

3. This fall, Shari is playing not only soccer but also field hockey.

3. Implicit double negatives: Some words imply a negative and therefore cannot be paired with an explicit negative. The words “barely,“ ”hardly,“ and ”scarcely” fall into this category.

1. Correct: Janie can hardly wait for vacation.

2. Incorrect: Janie can’t hardly wait for vacation.

Frequently Tested Prepositions

Idiomatic Expressions

Words That Can’t Pair with Negative Words

at

by

for

from

of

on

to

with

as . . . as

between . . . and

both . . . and

either . . . or

neither . . . nor

just as . . . so too

not only . . . but also

prefer . . . to

barely

hardly

scarcely

Commonly confused words

English contains many pairs of words that sound alike but are spelled differently and have different meanings, such as accept (to take or receive something that is offered) and except (with the exclusion of).

Other words, such as among (in a group of, or surrounded by, multiple things or people) and between (distinguishing one thing from one other thing), do not sound alike but have similar meanings that are often confused.

You’ll want to familiarize yourself with the following list of commonly misused words so you can spot them on test day.

Accept: to take or receive something that is offered

My niece accepted her pile of birthday gifts with great enthusiasm.

Except: with the exclusion of

All of the presents are toys except for a box containing a popular book series.

Affect: to act on, to have influence on something

The dreary, rainy weather negatively affected Rahul’s mood.

Effect: something that is produced by a cause; a consequence

A recent study explored the effects of weather on mental well-being.

Lay: to put or place something

My boss asked me to lay the report on her desk before I left for the day.

Lie: to rest or recline

After a long day of work, I just want to lie down on the couch.

Raise: to build or lift up something; to support the growth of someone

Many books are dedicated to the topic of raising children.

Rise: to get up

Ted likes to rise early in the morning to exercise before his children wake up.

Whose: a possessive pronoun

Whose uniform shirt is this?

Who’s: a contraction meaning “who is”

Who’s responsible for ordering new uniforms?

Their: a possessive pronoun for a plural noun or pronoun

The college students plan to travel internationally after their graduation.

They’re: a contraction for “they are”

They’re going to visit several countries in East Asia.

There: at a certain point or place

The students are excited to experience the foods and cultures there.

There’s: a contraction for “there is”

There’s a tour of an ancient palace that they’re looking forward to seeing.

Among: in a group of, or surrounded by, multiple things or people

Navya was among many doctoral candidates who visited the university.

Between: distinguishing one thing from one other thing

Navya had to decide between her top two doctoral program choices.

Amount: sum or quantity of multiple things that cannot be counted

The amount of pollution in the ocean is affecting dolphin populations.

Number: sum or quantity of a finite collection that can be counted

Scientists report that the number of dolphins has decreased significantly.

Less: a smaller extent or amount of things that cannot be counted

The common supermarket sign “10 items or less” is actually incorrect.

Fewer: of a smaller number, referring to things that can be counted

Since the items can be counted, the sign should read “10 items or fewer.”

Much: great in quantity, referring to things that cannot be counted

My sister has much more patience than I have.

Many: great in quantity, referring to things that can be counted

Many of her friends admire her ability to stay calm in difficult situations.

Good: satisfactory in quality, quantity, or degree; adjective

Dakota considered both the good and bad effects of wind energy before composing her essay.

Well: to perform an action in a satisfactory manner; adverb

Dakota wrote her essay so well that her professor used it as an example of excellent persuasive writing.

You need to do this:

· If the underlined portion includes a preposition, a conjunction, or barely/hardly/scarcely, look for a common idiom error.

· If the underlined segment includes a commonly misused word, check the context to determine whether it is used properly.

Explanation:

The underlined phrase contains the idiomatic expression not only . . . but also, so make sure it is correctly structured. (B) and (C) both omit part of the expression: (B) is missing “not only” and (C) is missing “but also.” (A) and (D) both use the correct wording, so analyze what the difference is between them. (D) places the word “to” before the beginning of the idiomatic phrase: “to not only.” However, the second part of the expression in (D) places the word “to” after the idiomatic phrase: “but also to.” Either placement would be acceptable, but the phrase must be structured consistently, so (D) is incorrect. (A) correctly structures the idiomatic expression: “not only to . . . but also to.”

If idioms give you trouble, study the information in this section and try these Drill questions before completing the Try on Your Own questions that follow. Edit each sentence to correct the idiom issue. Answers to the Drill can be found on the next page.

Drill

1. My book club meets at every third Sunday of the month.

2. Greyson made the gingerbread house by gumdrops, icing, candy buttons, and, of course, gingerbread.

3. The cheetah’s anatomy allows for extremely quick running speed not only due to its especially flexible spine but due to its unusually short skull.

4. After accidentally getting sunburned, I couldn’t hardly stand how itchy my skin felt.

5. Since she collected less pairs then her opponent, Candace lost the matching game.

Drill answers

Note: These are not the only ways to correct the sentences; your answers may differ.

1. My book club meets on every third Sunday of the month.

2. Greyson made the gingerbread house from gumdrops, icing, candy buttons, and, of course, gingerbread.

3. The cheetah’s anatomy allows for extremely quick running speed not only due to its especially flexible spine but also due to its unusually short skull.

4. After accidentally getting sunburned, I couldn’t hardly stand how itchy my skin felt.

5. Since she collected fewer pairs then her opponent, Candace lost the matching game.

Try on Your Own

Directions

Take as much time as you need on these questions. Work carefully and methodically. There will be an opportunity for timed practice at the end of the chapter.

1. Questions 38—44 refer to the following passage.

Hudson River School

Excepted by art historians as the first truly American art movement, the Hudson River School movement began in the early nineteenth century. The first works in this style were created by landscape painters Thomas Cole, Thomas Doughty, and Asher Durand, a trio of painters who worked during the 1820s in the Hudson River Valley and surrounding locations. Heavily influenced by European romanticism, these painters set out to convey the remoteness and splendor of the American wilderness. The strongly nationalistic tone of their paintings caught the spirit of the times, and within a generation, the movement had mushroomed to include landscape painters from all over the United States.

One factor contributing to the success of the Hudson River School was the rapid growth of American nationalism in the early nineteenth century. One affect of the War of 1812 was that it instilled Americans with a new sense of pride in their identity, and as the nation continued to grow, a desire grew to compete with Europe on both economic and cultural grounds. The vast panoramas of the Hudson River School fit the bill perfectly to providing a new movement in art, which was unmistakably American. The Hudson River School also arrived at a time when writers in the United States were turning their attention to the wilderness as a unique aspect of their nationality. The Hudson River School painters profited from this nostalgia because they effectively represented Americans’ perceptions of early America. Hardly not anyone questioned the view that the American character was formed by the frontier experience, and many writers wrote about their concerns regarding an increasingly urbanized country.

In keeping with this nationalistic spirit, even the painting style of the Hudson River School exhibited a strong sense of American identity. Unlike European painters, who brought to their canvases the styles and techniques of centuries past, the Hudson River School painters sought neither to embellish nor to idealize their scenes, but rather to portray nature with the objectivity and attention to detail of naturalists.

2.

1. NO CHANGE

2. Accepted

3. Excepting

4. Accepting

3.

1. NO CHANGE

2. with

3. in

4. due to

4.

1. NO CHANGE

2. effect

3. affecting

4. effecting

5.

1. NO CHANGE

2. by providing

3. in providing

4. only providing

6.

1. NO CHANGE

2. art, and

3. art that

4. art, so

7.

1. NO CHANGE

2. Not hardly anyone

3. Not anyone hardly

4. Hardly anyone

8.

1. NO CHANGE

2. sought to neither embellish nor to idealize their scenes

3. sought neither to embellish or to idealize their scenes

4. sought to neither embellish or to idealize their scenes

How Much Have You Learned?

Directions

For testlike practice, give yourself 9 minutes to complete this question set. Be sure to study the explanations, even for questions you got right. They can be found at the end of this chapter.

1. Questions 45—55 refer to the following passage.

Antarctica

To get some idea of what Antarctica is like, think of a place as remote as the far side of the Moon as strange as Saturn, and as inhospitable as Mars. A mere 2.4 percent of its 5.4 million square-mile landmass (50 percent larger than the United States) is ice-free, and that condition lasts for only a few months a year. Scientists estimate that 70 percent of the world’s fresh water is locked away in Antarctica’s ice cap, if this ice were ever to melt, sea levels might rise 200 feet, inundating coastal lands together with their major cities. In Antarctica, winds can blow at better than 200 mph, and temperatures can plummet as low as -128.6 degrees (Fahrenheit). There’s neither a town or a single tree, bush, or blade of grass on the entire continent.

Nevertheless, Antarctica is vital to life on Earth. The continent’s vast ice fields reflect sunlight back into space, preventing the planet from overheating. The cold water that the icebergs generate flows north and mixes with equatorial warm water, producing currents and clouds that create ultimate, complex weather patterns. Antarctic seas teem with life—from microscopic phytoplankton and tiny krill at the bottom of the food chain to killer whales and leopard seals at the top giving these waters a vital status among the Earth’s ecosystems. Unique species of birds and mammals make their homes in the frigid waters of the Southern Ocean that lap the continent’s edge, some found nowhere else on the planet.

The relative inaccessibility and near pristine state of Antarctica makes it an invaluable place for scientific research today. Clues to ancient climates lie buried deep in layers of Antarctic ice—clues such as trapped bubbles of atmospheric gases, which can help scientists draw a better picture of what Antarctica was like in the past. Until recently, most scientists thought that Antarctica has been covered by ice for 40 million to 52 million years and that the present ice cap is about 15 million years old. However, the discovery of remnants of a beech forest near the head of the Beardmore glacier, approximately 250 miles from the South Pole, provides evidence that Antarctica may have been both ice-free and much more temperate 2.5 million to 5 million years ago than it is now. Similar fossil finds made elsewhere suggest that western Antarctica was perhaps completely ice-free as recently as 100,000 years ago and scientists, as a result, are conducting new research to enhance their understanding of Antarctica’s climate changes.

2.

1. NO CHANGE

2. Moon, as strange as Saturn,

3. Moon as strange Saturn

4. Moon, strange as Saturn,

3.

1. NO CHANGE

2. Antarctica’s

3. their

4. it’s

4.

1. NO CHANGE

2. ice cap; and if

3. ice cap and if

4. ice cap; if

5.

1. NO CHANGE

2. Their’s

3. There are

4. They’re

6.

1. NO CHANGE

2. either a town or a single

3. no town nor a single

4. neither a town nor a single

7.

1. NO CHANGE

2. ultimately complexly create

3. ultimately create complex

4. create

8.

1. NO CHANGE

2. life, from microscopic phytoplankton and tiny krill at the bottom of the food chain to killer whales and leopard seals at the top; giving

3. life; from microscopic phytoplankton and tiny krill at the bottom of the food chain to killer whales and leopard seals at the top—giving

4. life, from microscopic phytoplankton and tiny krill at the bottom of the food chain to killer whales and leopard seals at the top, giving

9.

1. NO CHANGE

2. Some found nowhere else on the planet, the frigid waters of the Southern Ocean that lap the continent’s edge are home to unique species of birds and mammals.

3. Unique species of birds and mammals, some found nowhere else on the planet, make their homes in the frigid waters of the Southern Ocean that lap the continent’s edge.

4. The frigid waters of the Southern Ocean that lap the continent’s edge, some found nowhere else on the planet, are home to unique species of birds and mammals.

10.

1. NO CHANGE

2. make

3. made

4. is making

11.

1. NO CHANGE

2. were providing

3. provide

4. providing

12.

1. NO CHANGE

2. ago so scientists

3. ago, scientists

4. ago. Scientists

13.

Reflect

Directions: Take a few minutes to recall what you’ve learned and what you’ve been practicing in this chapter. Consider the following questions, jot down your best answer for each one, and then compare your reflections to the expert responses on the following page. Use your level of confidence to determine what to do next.

Name at least three ways to correct a run-on sentence.

How does the PSAT test subject-verb agreement and parallelism?

What are the three different pronoun forms? When do you use each one?

What is the difference between an adjective and an adverb?

What are the three ways that apostrophes are tested on the PSAT?

Which commonly confused words do you need to be especially careful to look out for?

EXPERT RESPONSES

Name at least three ways to correct a run-on sentence.

There are a number of ways to fix a run-on sentence on the PSAT. The six ways that you are likely to see are: 1) use a period to create two separate sentences, 2) use a semicolon between the two independent clauses, 3) use a colon between the two independent clauses, 4) make one one clause dependent, 5) add a FANBOYS conjunction after the comma, or 6) use a dash between the two independent clauses.

How does the PSAT test subject-verb agreement and parallelism?

A subject and verb must always agree in person (first, second, or third) and number (singular or plural). You will need to be able to spot subject-verb mismatches and correct them. Parallelism requires that all items in a list, a compound, or a comparison are in parallel form. The PSAT may test lists or comparisons in which one item is in the wrong form.

What are the three different pronoun forms? When do you use each one?

The three forms are subjective (when the pronoun is the subject), objective (when the pronoun is the object of a verb or preposition), and possessive (when the pronoun expresses ownership).

What is the difference between an adjective and an adverb?

An adjective is a single word that modifies a noun or a pronoun, while an adverb is a single word that modifies a verb, an adjective, or another adverb.

What are the three ways that apostrophes are tested on the PSAT?

Apostrophes on the PSAT are used to: 1) indicate the possessive form of a singular noun (’s), 2) indicate the possessive form of a plural noun (s’), or 3) indicate a contraction (don’t = do not).

Which commonly confused words do you need to be especially careful to look out for?

The answer to this question is specific to you. If you have concerns about more than half of the words out of the list of 24 provided in the “Agreement: Idioms” lesson, consider making flash cards to help you practice. The extra effort will ensure that you do not confuse any of the commonly confused words on test day.

NEXT STEPS

If you answered most questions correctly in the “How Much Have You Learned” section, and if your responses to the Reflect questions were similar to those of the PSAT expert, then consider sentence structure, punctuation, and agreement areas of strength and move on to the next chapter. Come back to these topics periodically to prevent yourself from getting rusty.

If you don’t yet feel confident, review those parts of this chapter that you have not yet mastered. In particular, review punctuation usage in the “Sentence Structure: The Basics” and “Sentence Structure: Commas, Dashes, and Colons” lessons, as well as how to select the appropriate pronoun or modifier in the “Agreement: Pronouns” and “Agreement: Modifiers” lessons. Then try the questions you missed again. As always, be sure to review the explanations closely.

Answers and Explanations

1. Review the Explanation portion of the Sentence Structure: The Basics lesson.

2. C

Difficulty: Medium

Getting to the Answer: As written, the first sentence is a fragment: it does not express a complete thought. Eliminate (A). Also eliminate (D), since a semicolon joins two independent clauses. (B) is incorrect because it uses the conjunction “and” incorrectly, turning the entire sentence into a fragment. (C) is correct because it uses a comma to join the dependent clause beginning “Since” to the independent clause “he noticed . . .”

3. B

Difficulty: Medium

Getting to the Answer: As written, the sentence uses a semicolon to join the sentence’s parts. The first part of the sentence is an independent clause with subject-verb “He decided.” The second part does not contain a subject or express a complete thought, so eliminate any answer choices that punctuates the sentence as if it contains two independent clauses: (A), a semicolon; (C), a comma and FANBOYS conjunction; and (D), a period that results in two sentences. (B) is correct because the word “and” joins the compound verb “decided . . . and found”; no extra punctuation is necessary.

4. D

Difficulty: Medium

Getting to the Answer: The part of the sentence before the underline is an independent clause; it expresses a complete thought with subject “Some people” and verb “are.” The part after the underline, however, is a dependent clause because it does not express a complete thought: “Though the number . . . ” The word “though” subordinates the second clause. You need to find a way to join the independent clause to the subordinate (dependent) clause that follows it. (D) is correct because a comma is the appropriate way to join an independent clause and a dependent, subordinate clause. (A) and (C) are incorrect because they punctuate the sentence as though the second part were an independent clause. (B) is incorrect because it creates a run-on.

5. D

Difficulty: Hard

Getting to the Answer: The underlined section contains part of a long list of side effects. Normally, you would separate items in a list with commas, but when a list contains sublists, the sublists must be separated by semicolons and the items within them must be separated by commas. (D) is correct because it correctly punctuates the list by using commas between the items in the sublists of side effects and semicolons between the distinct groups of side effects.

6. B

Difficulty: Medium

Getting to the Answer: The parts of the sentence before and after the underline are both independent clauses. Each expresses a complete thought and has a subject and predicate verb (“problem . . . is” and “these bacteria cannot be”). Eliminate any answer choices that are not ways to combine two independent clauses. (A) incorrectly uses a comma. (C) is incorrect because the lack of any punctuation between independent clauses results in a run-on. (D) is incorrect because the word “although” turns the second clause into a subordinate clause, so the clauses cannot be joined by a semicolon. (B) is correct because it joins two independent clauses with a semicolon.

1. Review the Explanation portion of the Sentence Structure: Commas, Dashes, and Colons lesson.

2. C

Difficulty: Easy

Getting to the Answer: As written, the colon is used improperly: a colon can follow an independent clause only. Eliminate (A). The words before the punctuation mark, “A practical man,” form an introductory phrase. An introductory phrase should be followed by a comma, so (C) is correct. (B) is incorrect because it runs the phrases together, and the use of the dash in (D) is incorrect for the same reason as is the colon in (A).

3. D

Difficulty: Hard

Getting to the Answer: This sentence is complicated, so break it down into its component parts so you can determine which punctuation is correct. The subject of the sentence is “He” and the verb is “would . . . accept.” The phrases after “layers” are prepositional phrases that describe the layers. The underline appears in one of these prepositional phrases: “from candles, and incense burned during religious functions.” Punctuation should not separate a preposition from its object, even when the preposition (“from”) has a compound object (“candles and incense”). Therefore, eliminate any choices that separate the preposition from the objects. (D) is the only one that doesn't separate them, so it is correct.

4. D

Difficulty: Medium

Getting to the Answer: This underline contains two nouns (“solvents” and “equipment”) and their descriptive adjectives. When more than one adjective modifies a noun, there should be commas between the adjectives only if you could change the order of the adjectives without changing the meaning. For the first noun here, “solvents,” the word “cleaning” specifies the type of solvents. Thus, “special” is actually modifying “cleaning solvents.” You cannot change the order of the adjectives without changing the meaning—“cleaning special solvents” does not make sense. Therefore, eliminate (A), which adds a comma between “special” and “cleaning.” Perform the same analysis on “equipment.” Switching the order of the adjectives would result in an illogical phrase (“analysis computerized equipment”), so “analysis” specifies the type of “equipment,” and no comma is necessary. Eliminate (B). There is no reason to separate the nouns “solvents” and “equipment” with a comma as in choice (C), so (D) is correct.

5. B

Difficulty: Medium

Getting to the Answer: Remember that punctuation must be used to set off parenthetical information. In this sentence, the phrase “as they claimed” is parenthetical in the sentence— without this nonessential phrase, the essential idea of the sentence, “the restorers did not achieve this effect merely by removing the dirt . . . from the frescoes” still makes logical sense. Since punctuation must set off parenthetical information, (B) is correct because it is the only choice that uses a comma at both the beginning and the end of the phrase.

6. A

Difficulty: Medium

Getting to the Answer: Analyze both the types of clauses and the relationship between their ideas to determine how they should be punctuated. The clause before the punctuation is an independent clause, a complete thought with subject-verb: “the restorers did not achieve.” The clause after the punctuation is also an independent clause with subject-verb: “the team removed.” The ideas of the clauses are closely related: the first sentence states that the “effect” was not “merely” the result of “removing the dirt and animal glue,” and the second sentence explains what else was done to achieve the effect (“remov[ing] Michelangelo’s final touches”). Since the part of the sentence before the punctuation is an independent clause and the second part helps to explain the first, a colon effectively punctuates these two closely related clauses, so (A) is correct. The other choices create run-on sentences by improperly joining two independent clauses.

7. A

Difficulty: Easy

Getting to the Answer: Items in a series should be separated by commas. This series is lengthy, but the rule is the same. The underlined portion lists the things that the “solvents” did: “stripped,” “reacted,” and “produced.” These should be separated by commas, so (A) is correct. Semicolons, as in (C) and (D), are required only in series to separate groups of related items; when this occurs, the related items are separated by commas.

8. B

Difficulty: Medium

Getting to the Answer: This sentence begins with a subordinating clause (“Since the layers . . . protection”). Because this clause is not a complete thought, it is a dependent clause that must be joined to the rest of the sentence with a comma, so (B) is correct. (A) runs the clauses together, while (C) and (D) punctuate the sentence as though it consisted of two independent clauses.

9. B

Difficulty: Medium

Getting to the Answer: The phrase “an extensive climate-control system” is parenthetical to the meaning of the sentence. Although it provides clarifying information about what “the measure” is, the sentence would still be logical even if the phrase were removed. Parenthetical information must be set off by a pair of matching punctuation marks, such as commas or dashes, so (B) is correct. (A) and (C) are incorrect because they include mismatched punctuation marks around the phrase. (D) is incorrect because semicolons cannot be used to set off parenthetical information.

1. Review the Explanation portion of the Agreement: Verbs lesson.

2. A

Difficulty: Medium

Getting to the Answer: Make sure that the underlined verb matches the tense of the surrounding context and agrees with its subject. The verb should be in the present tense since it refers to the “current image,” so eliminate (B) and (C). The subject of the verb “capture” is the compound subject, “The woolly mammoth and the mastodon.” Since a compound subject is treated as a plural, the singular verb in (D) is incorrect. Only (A) is left and is correct because it has the correct tense and agrees with its subject.

3. C

Difficulty: Medium

Getting to the Answer: Check whether the subject of the underlined verb is singular or plural. Finding the subject is complicated by the fact that a prepositional phrase (“to the modern-day elephant”) separates the verb from its subject, “precursors.” This subject is plural, so eliminate the singular verbs in (A) and (D). You know that these animals are “now-extinct,” so you need a past tense verb; eliminate (B). Only (C) is left and is correct.

4. B

Difficulty: Medium

Getting to the Answer: The verb’s subject is the singular “team,” and the action happened in the past, as indicated by verbs “descended” and “helped” in the surrounding sentences. Eliminate (C), which is not in the past tense, and (D), which is a plural verb form. One of the remaining choices, (A), is a verb form that indicates an action that occurred before another past action, “had completed.” This action did not occur before another action in the sentence, but rather occurred “after” the other events described. Thus, the simple past tense, (B), is correct.

5. D

Difficulty: Medium

Getting to the Answer: Comparisons on the PSAT must be written in parallel form. This comparison identifies two “scientific theories”: “meteor showers” and “massive volcanic eruptions.” Be very careful that the wording you choose directly compares these two ideas. The comparison begins after the word “ranged” and fits into the pattern “from . . . to . . .” So, as written, the sentence compares “meteor showers” to “suggestions of massive volcanic eruptions.” This comparison is not parallel, so eliminate (A). (B) and (C) make similar errors, comparing the “meteor showers” to “suggesting” and “theories,” respectively. (D) completes the comparison logically.

6. B

Difficulty: Medium

Getting to the Answer: This underlined verb has the subject “community.” Although this noun refers to numerous people, the collective noun itself is treated as singular. Eliminate any answer choices that use a plural verb: (A) and (D). Using the -ing form of the verb, as in (C), results in a sentence fragment. (B) appropriately uses a present-tense, singular verb and is therefore correct.

7. D

Difficulty: Easy

Getting to the Answer: Use context to determine the appropriate tense for the underlined verb. Although the scientists’ action, “consider,” is in present tense, the verbs referring to the mammoths, which “died out” according to the previous sentence, must be in past tense. Only (D) is in the past tense, so it is correct.

1. Review the Explanation portion of the Agreement: Pronouns lesson.

2. C

Difficulty: Medium

Getting to the Answer: When a pronoun is underlined, make sure it agrees with its antecedent and is in the correct form based on its placement in the sentence. The underlined pronoun refers to people (“authors”), not things, so eliminate the pronoun “which,” choice (B). The subjective form of the pronoun is required since the pronoun is part of the subject of the verb “were.” Therefore, the subjective pronoun “who,” choice (C), is correct.

3. D

Difficulty: Medium

Getting to the Answer: An underlined pronoun must match the pronouns used in the surrounding context. Determine who the writer is referring to in saying that these authors “compelled” someone. In surrounding sentences, the writer uses the pronouns “we” and “ours” to refer to those impacted by these authors; the writer thus includes both the reader and herself in this designation. This pronoun should include the same groups, so the first-person plural “us,” (D), is correct. (B) is incorrect because it includes only the writer, which does not match the other surrounding pronouns. (A) is incorrect because it is in the second person, and (C) is incorrect because it is in the third person, neither of which is used elsewhere in the passage.

4. C

Difficulty: Medium

Getting to the Answer: The pronoun “they” sounds correct in this usage, and the writer uses this pronoun in the previous sentence to refer to the authors, but carefully check to make sure this underlined pronoun has a clear antecedent. The pronoun is meant to refer to the authors, as indicated later in the sentence by the reference to “their writings.” However, the nearest antecedent for the pronoun “they” is “readers” in the previous sentence. Even though you may understand the intended meaning, the pronoun here is ambiguous and thus incorrect. (C) is correct because it is the only choice that clarifies the subject.

5. B

Difficulty: Medium

Getting to the Answer: Determine the antecedent of the underlined pronoun. The pronoun is possessive, indicating that the “readings” of Ulysses belongs to someone. The sentence previously refers to the plural “literary critics,” so this pronoun should be plural as well; eliminate (C). Although the writer clearly read Ulysses and refers to her opinion about it in the remainder of the paragraph, these readings belong only to other literary critics, so eliminate (A) and (D). (B) is correct.

6. C

Difficulty: Medium

Getting to the Answer: The underlined pronoun is possessive, indicating that the “book” either belongs to or was written by someone. The antecedent is unclear, as the sentence previously refers to The Odyssey, written by Homer, and the Odyssey’s “hero.” However, the context indicates that the “book” referred to in this case is the “opposite” of The Odyssey, so it must be the book written by Joyce. Although you can logically determine the pronoun’s antecedent, the writer should eliminate the grammatical ambiguity by using Joyce’s name rather than a pronoun, so (C) is correct.

7. A

Difficulty: Medium

Getting to the Answer: Besides matching their antecedent, pronouns must also be in the correct form: subjective, objective, or possessive. The underlined pronoun should be in the objective form since it is the object of the preposition “of.” (A) is correct. (B) and (D) are incorrect because they use pronoun forms that are not appropriate for the context. (C) is incorrect because it is not a grammatically correct pronoun form.

1. Review the Explanation portion of the Agreement: Modifiers lesson.

2. A

Difficulty: Medium

Getting to the Answer: The answer choices contain frequently confused versions of possessive pronouns and contractions, so carefully assess the context of the sentence to determine which is correct. A possessive pronoun is appropriate because the sentence is referring to the “mediumthat belongs to thenovel,” so eliminate the contractions “it’s” and “they’re,” which mean it is and they are, respectively. The “novel” is singular, so the singular possessive pronoun “its,” (A), is correct.

3. C

Difficulty: Hard

Getting to the Answer: The placement of a modifying phrase affects the meaning of a sentence, so make sure every phrase is placed correctly. The writer intends to use the modifying phrase “becoming cruel, shortsighted, and constantly dissatisfied with real life” to describe Emma; however, as written, the phrase describes her “friends and family.” Eliminate (A). Only (C) places the modifying phrase next to “Emma,” so it is correct.

4. B

Difficulty: Hard

Getting to the Answer: The underlined words modify the noun “fashion.” Unless one of the adjectives is a defining modifier of the noun (as in the phrase “red dump truck,” which could not be written “dump red truck”), two adjectives cannot be placed together without a comma between them, so (A) and (C) are incorrect. Only (B) uses the modifiers correctly, the adverb “gloriously” describing the adjective “dramatic” and the adjective “dramatic” describing the noun “fashion.” (D) is incorrect because it uses an adverb, “dramatically,” to modify a noun, “fashion.”

5. B

Difficulty: Medium

Getting to the Answer: Sometimes the trickiest modifier placement questions concern the shortest words, such as “only.” In such cases, carefully analyze the writer’s intended meaning. The writer is comparing Emma with the other characters in the novel, making the point that no other character “habitually reads for pleasure.” Therefore, it is Emma who is “the only character” who reads in this way, so (B) is correct. (A) is incorrect because it unnecessarily repeats the modifier “only.” (C) and (D) both change the intended meaning. (C) singles out “novel,” as though more than one novel were being discussed. (D) implies that Emma only does one thing: “habitually reads for pleasure.”

6. A

Difficulty: Easy

Getting to the Answer: When you see an underlined comparison, determine how many things are being compared. The underlined modifier compares “the characters making up the novel’s non-reading population” with “Emma” in regard to social responsibility. Since only two groups are part of the comparison, a comparative word is appropriate, so (A), “more,” is correct. (B) and (C) both result in an improperly worded comparison, and (D) is incorrect because “most” is a superlative, used when comparing three or more items.

7. A

Difficulty: Medium

Getting to the Answer: This question concerns correct apostrophe placement in a possessive noun. You know the underlined word must be in a possessive form because the sentence is referring to the “unwillingness” of the characters; eliminate (C). There are many other characters in Madame Bovary—her husband, her household, and her family are all mentioned in the passage. (A) is correct because it is the correct way to punctuate a plural possessive: place the apostrophe immediately after the letter that makes the word plural. (B) is incorrect because it is a singular possessive. (D) is not written with the correct structure for a plural possessive.

1. Review the Explanation portion of the Agreement: Idioms lesson.

2. B

Difficulty: Easy

Getting to the Answer: This question is testing two commonly confused words: accept and except. Except denotes an exclusion, while accept can mean to either receive or recognize something. In this context, “art historians” are recognizing the status of an art movement, so eliminate any choice containing “except”: (A) and (C). The -ed form of the word fits in this context, so (B) is correct.

3. A

Difficulty: Easy

Getting to the Answer: You need to identify the correct preposition for the idiom influenced ____ something. Since the correct idiom is influenced by, (A) is correct.

4. B

Difficulty: Easy

Getting to the Answer: This question is testing two commonly confused words: affect and effect. Affect means to influence and is most often used as a verb. While effect can be used as a verb (meaning to bring something about, as in “effected a solution”), it is usually used as a noun meaning result. In this sentence, the word is used as a noun (try substituting the word result—if you can, you can use effect), so effect must be correct. Adding the -ing ending does not make sense in context, so (B) is correct.

5. B

Difficulty: Easy

Getting to the Answer: The correct preposition can often only be determined in the context of the sentence. This sentence states that the Hudson River School was suited to the task of competing with Europe. The function of the prepositional phrase is to explain how that was accomplished, and the phrase “by providing” best serves that function, so (B) is correct.

6. C

Difficulty: Hard

Getting to the Answer: This question is testing two commonly confused words: that and which. That is used with essential, or defining, information. Which is used with nonessential, or parenthetical, information. In this case, that is correct because it is necessary to specify that the “art” is “unmistakably American.” The phrase “art, which was unmistakably American” indicates that all art is unmistakably American, which is clearly not the intended meaning. (C) is correct. (B) and (D) result in ungrammatical sentences.

7. D

Difficulty: Medium

Getting to the Answer: While explicit double negatives are easier to catch and avoid, you must also be on the lookout for implicit double negatives. Hardly is a word that cannot be directly paired with a negative, such as not. The only answer choice that avoids an implicit double negative construction is (D), so it is correct.

8. A

Difficulty: Medium

Getting to the Answer: This question requires you to select the correct construction of the idiomatic expression neither . . . nor. (C) and (D) use the phrasing “neither . . . or,” so eliminate them. The preposition “to” must be consistently placed in the expression. As written, the word “to” appears after each word in the expression: “neither to embellish nor to idealize,” so (A) is correct. (B) incorrectly places the word “to” both before and within the phrase: “to neither embellish nor to idealize.”

1. B

Difficulty: Easy

Category: Sentence Structure: Commas, Dashes, and Colons

Getting to the Answer: The sentence creates a series: attributes of locations in space that help the reader picture Antarctica. Each item in the series is worded “as (adjective) as Moon/Saturn/Mars.” Basic series should be punctuated with commas between each of the items, so (B) is correct. (A) and (C) each omit necessary commas from the series, and (D) incorrectly omits the word “as” in the second series item.

2. B

Difficulty: Hard

Category: Agreement: Pronouns

Getting to the Answer: Always check underlined pronouns for ambiguity. In this case, the antecedent of the pronoun is unclear: although the writer is clearly describing qualities of Antarctica, the nearest antecedent is “Mars.” After eliminating the pronouns that are ambiguous, you’re left with (B), which correctly clarifies the intended location. Note that even if a pronoun in this location were not ambiguous, (C) would be incorrect because it is plural and (D) would be incorrect because it is the contraction of “it is.”

3. D

Difficulty: Medium

Category: Sentence Structure: The Basics

Getting to the Answer: The underlined comma separates two independent clauses, each expressing a complete thought with a subject and predicate verb (“Scientists estimate” and “sea levels might rise”). Remember that semicolons combine two independent clauses, while commas can join independent clauses only when followed by a FANBOYS conjunction. Choice (D) is correct because it properly joins the independent clauses with a semicolon. The other answer choices are not valid ways to join independent clauses. (A) creates a run-on because it joins independent clauses with only a comma. (B) adds an unnecessary “and” after the semicolon. (C) omits the necessary comma before “and.”

4. A

Difficulty: Medium

Category: Agreement: Idioms

Getting to the Answer: The word their and theirs are possessive pronouns, but “their’s” is never correct, so eliminate (B). The singular “a town” that follows the underlined portion for this question requires a singular verb. Only (A) offers the singular verb “is” and is therefore correct. The remaining choices are incorrect because they use the plural verb “are” either outright as (C) does, or hidden in a contraction, as in (D), where “they’re” is a shortened version of “they are.”

5. D

Difficulty: Medium

Category: Agreement: Idioms

Getting to the Answer: The underlined phrase contains an idiomatic expression. The writer is identifying things that are not in Antarctica, not expressing that the continent could logically only have either a town or vegetation, so eliminate (B). The correct idiomatic expression is “neither . . . nor.” Only correct answer (D) uses this structure.

6. C

Difficulty: Medium

Category: Agreement: Modifiers

Getting to the Answer: When modifiers are underlined, make sure they are in the correct format to modify the intended word. The underlined modifiers are in the last part of a sentence that describes the impact of Antarctic water. The modifier “ultimate” is not intended to modify “weather patterns,” as written, but to indicate that the creation of these weather patterns is the final part of a process; eliminate (A). Only (C) structures the modifiers so that “ultimately” modifies the verb “create” and “complex” modifies “weather patterns,” so (C) is correct. (D) is incorrect because it alters the original intended meaning of the sentence.

7. D

Difficulty: Hard

Category: Sentence Structure: Commas, Dashes, and Colons

Getting to the Answer: You must determine what punctuation is correct around this long phrase that appears in the middle of a sentence. As written, the descriptive phrase that begins “giving these waters” confusingly runs into the previous part of the sentence, so eliminate (A). The underlined prepositional phrases are best viewed as parenthetical information that describes the type of life in Antarctic seas. If the phrases were removed, the sentence would still stand on its own: “Antarctic seas teem with life, . . . giving these waters a vital status among the Earth’s ecosystems.” Look for an answer choice that punctuates the parenthetical information appropriately, using either a pair of commas or dashes to set off the nonessential phrase. (D) does this with commas and is correct. (B) and (C) are incorrect because the use of a semicolon would indicate that two independent clauses are being joined, but each creates one dependent clause.

8. C

Difficulty: Medium

Category: Agreement: Modifiers

Getting to the Answer: The answer choices place the descriptive phrase “some found nowhere else on the planet” in different locations. The phrase should be placed near “unique species of birds and mammals,” since it describes these living creatures rather than “frigid waters” or “the continent’s edge.” Only correct answer (C) places these phrases together.

9. B

Difficulty: Medium

Category: Agreement: Verbs

Getting to the Answer: Check underlined verbs for tense consistency and agreement with their subjects. The context of the paragraph makes it clear that research in Antarctica occurs “today,” so eliminate (C), which confines the action to only the past. To determine which present tense verb is correct, find the verb’s subject. Although the noun “Antarctica” is nearest, the subject is actually the compound “relative inaccessibility and near pristine state.” Compound subjects with and should be treated as plural, so the plural verb “make,” (B), is correct.

10.A

Difficulty: Medium

Category: Agreement: Verbs

Getting to the Answer: Determine the subject of the underlined verb so you can ensure the subject and verb are in agreement. Although there are many words in between, the subject of the verb is “discovery”; all the words in between are prepositional or descriptive phrases that describe the discovery or its location. Since “discovery” is singular, the singular “provides,” (A), is correct. (B) and (C) are incorrect because they are plural verbs. (D) is incorrect because it would result in a sentence fragment.

11.D

Difficulty: Easy

Category: Sentence Structure: The Basics

Getting to the Answer: A sentence is a run-on if it has two improperly joined independent clauses that could be stand-alone sentences. The parts of the sentence before and after the underline are both independent clauses that express complete thoughts. Both a comma and a FANBOYS conjunction are required when combining two independent clauses. Eliminate (A) and (B) because they are missing the comma, and eliminate (C) because it is missing a FANBOYS conjunction. Choice (D) correctly divides the two independent clauses into two separate sentences.